You are on page 1of 6

Test 2

201900 MA112 Calculus II (Mode: F) SOLUTION

Instructions: This is strictly a supervised, unrehearsed hand–written test. You have at most 2
hours to answer ALL nine questions. Do not use pencil or red pen when writing your answers.
Write legibly. Maximum mark is 45. Some useful information are also included at the end of
the questions.

Z
Q1. Evaluate tan6 x sec4 x dx.

Hint: tan2 x + 1 = sec2 x

Solution.
Z Z Z
6 4
tan x sec x dx = tan x(sec x) dx = tan6 x sec2 x sec2 x dx
6 2 2

Z Z
= tan x(1 + tan x) sec x dx = u6 (1 + u2 ) du
6 2 2
(u = tan x)
1 7 1 9 1 1
= u + u + C = tan7 x + tan9 x + C
7 9 7 9
,

[4 marks]
Z
1
Q2. With a suitable trigonometric substitution, evaluate √ dx. You must express
x2 x2 + 4
your answer in terms of x.
Hint: x = a tan θ

Solution. Let x = 2 tan θ, − π2 < θ < π2 , dx = 2 sec2 θ dθ. Then


Z Z Z
1 2 sec2 θ dθ 2 sec2 θ dθ
√ dx = √ =
x2 x2 + 4 4Z tan2 θ 4 tan2 θZ + 4 4 tan2 θ · 2 sec θ
Z
1 sec θ 1 cos θ 1 du
= dθ = dθ = (u = sin θ)
4 tan2 θ 4 sin2 θ 4 u2

1 1 csc θ x2 + 4
= − +C=− +C=− +C=− + C.
4u 4 sin θ 4 4x
,

[4 marks]
Z
7x + 5
Q3. Use the method of partial fractions to evaluate dx.
x2 + 2x − 3

Set: 28 Jan 2019 Page 1 of 6 RSH20190130095600

Lautoka Campus
Test 2
201900 MA112 Calculus II (Mode: F) SOLUTION

Solution. First notice that


7x + 5 7x + 5 A B
= = + ;
x2 + 2x − 3 (x − 1)(x + 3) x−1 x+3
that is,
7x + 5 = A(x + 3) + B(x − 1).
When x = 1, we have 12 = 4A ⇒ A = 3. When x = −3, we have −16 = −4B ⇒ B = 4.
Therefore,
Z Z
7x + 5 3 4

dx = + dx = 3 ln |x − 1| + 4 ln |x + 3| + C
x2 + 2x − 3 x − 1 x + 3
= ln (x − 1)3 (x + 3)4 + C.

[4 marks]
Z2
1
Q4. Let I = dx. How large should we take n in order to guarantee that the Simpson’s
1 x
Rule approximation for I is accurate to within 0.0001?

Solution. In [1, 2],


(4) 24 24 24
|f (x)| = 5 = 5 ≤ 5 = 24.
x x 1
Let K4 ≡ 24. In Sn, we need
r
(2 − 1)5 24 24 4 1 24
|ES | ≤ 4
= < 0.0001 ⇔ n > · = 6.042750795.
180n 180n4 0.0001 180
Take n = 8 (being even). ,

[3 marks]
Z∞
1
Q5. Show that the improper integral dx = π.
−∞ 1 + x2
Note: arctan θ → ± π2 as θ → ±∞

Solution. We have
Z∞ Z0 Z∞
1 1 1
dx = dx + dx
−∞ 1 + x2 −∞ 1+x 0 1 + x2
It now follows that
Z0 Z0 0
1 dx −1

−1 −1

dx = lim = lim tan x = lim tan (0) − tan t
−∞ 1 + x t→−∞ t 1 + x2 t→−∞ t t→−∞
π π
 
= 0− − =
2 2

Set: 28 Jan 2019 Page 2 of 6 RSH20190130095600

Lautoka Campus
Test 2
201900 MA112 Calculus II (Mode: F) SOLUTION

Similarly,
Z∞ Zt t
1 dx −1

−1 −1

dx = lim = lim tan x = lim tan t − tan 0
0 1+x t→+∞ 0 1 + x2 t→+∞ 0 t→+∞
π π
= −0= .
2 2
Therefore, Z∞
1 π π
2
dx = + = π.
−∞ 1+x 2 2
,

[4 marks]
Q6. (a) Solve the initial–value problem
dy 2t + 1
= , y(0) = −1
dt 2y − 2
by separating the variables then integrate. (You may leave your general solution in
implicit form.)
Solution. We separate the variables then integrate.
dy 2t + 1
=
dt 2y − 2
Z Z
(2y − 2) dy = (2t + 1) dt

y(y − 2) = t2 + t + C

Thus, the general solution in implicit form is

y(y − 2) = t2 + t + C.

(We write the general solution in implicit form because we can’t make y the subject
so we leave it as like that.) Considering the initial condition y(0) = −1, i.e. y = −1
when t = 0, we have

−1(−1 − 2) = 02 + 0 + C ⇒ C = 3.

Hence, the particular solution of the initial–value problem is

y(y − 2) = t2 + t + 3.

,
(b) Write the following differential equation in standard form and use the method of
integrating factor to find the general solution.
dy
x + 2y = 10x2
dx

Set: 28 Jan 2019 Page 3 of 6 RSH20190130095600

Lautoka Campus
Test 2
201900 MA112 Calculus II (Mode: F) SOLUTION

Solution. The standard form is


dy
+ p(x)y = q(x).
dx
We divide through the given differential equation by x to get the standard form:
dy 2
+ y = 10x; (1)
dx x
2
p(x) = x
and q(x) = 10x. The integrating factor is
R R 2 2
µ=e p(x) dx
=e x dx
= e2 ln |x| = eln x = x2 .
Multiply through (1) by µ to get
dy
x2 + 2xy = 10x3
dx
d h 2 i
x y = 10x3
Z dx Z
d h 2 i
xy = 10x3 dx
dx
Z !
x4
x y = 10 x3 dx = 10
2
+ C1
4
!
10 x4 5 2 C
y = + C1 = x + 2,
x2 4 2 x
where C = 10C1 . ,

[4+4=8 marks]
p
Q7. (a) Describe the domain of f(x, y, z) = 1 − x2 − y2 − z2 .
Solution. The domain of f is
dom(f) = {(x, y, z) : 1 − x2 − y2 − z2 ≥ 0} = {(x, y, z) : x2 + y2 + z2 ≤ 1}
which contains all the point on and inside the sphere x2 + y2 + z2 = 1 centred at the
origin with radius 1. ,
(b) Let f(x, y) = sin(x2 + y2 ). Find: fxy , fyx , fxx (1, 0).
Solution. Firstly, note that fx = 2x cos (x2 + y2 ) and fy = 2y cos (x2 + y2 ). Then
 
fxy = −4xy sin x2 + y2 ;
   
fxx = 2 cos x2 + y2 − 4x2 sin x2 + y2 ;
fxx (1, 0) = 2cos(1 + 0) − 4(1) sin(1 + 0) = 2 cos 1 − 4 sin 1;
 
fyx = −4xy sin x2 + y2 .

Set: 28 Jan 2019 Page 4 of 6 RSH20190130095600

Lautoka Campus
Test 2
201900 MA112 Calculus II (Mode: F) SOLUTION

[2+3=5 marks]

2n
Q8. Consider the sequence {an } = .
3n + 2
an+1
(a) Use the ratio to determine if the sequence {an } is strictly increasing or decreasing.
an
Solution. We have
an+1 2(n + 1) 3n + 2 (2n + 2)(3n + 2)
= · =
an 3(n + 1) + 2 2n (3n + 5)(2n)
2
(6n + 10n) + 4 4
= 2
=1+ 2 > 1 for all n ≥ 1.
6n + 10n 6n + 10n
Thus
an+1
> 1 ⇒ an+1 > an for all n ≥ 1
an
and therefore {an } is strictly increasing. ,
(b) Use the difference an+1 − an to determine if the sequence {an } is strictly increasing or
decreasing.
Solution. We have
2n + 2 2n 4
an+1 − an = − = 2
> 0 for all n ≥ 1.
3n + 5 3n + 2 9n + 21n + 10
Thus
an+1 − an > 0 ⇒ an+1 > an for all n ≥ 2
and therefore {an } is strictly increasing. ,
[2+2=4 marks]
Q9. Let {an } be the sequence defined recursively by
12
a1 = −1 and an+1 = for n ≥ 2.
4 + an
(a) Write the first five terms of the sequence.
Solution.
a1 = −1,
12
a2 = = 4,
4−1
12 12 3
a3 = = = = 1.5,
4+4 8 2
12 24
a4 = = = 2.1818181,
4 + 23 11
12 132 33
a5 = 24
= = = 1.94118
4 + 11 68 17
,

Set: 28 Jan 2019 Page 5 of 6 RSH20190130095600

Lautoka Campus
Test 2
201900 MA112 Calculus II (Mode: F) SOLUTION

(b) Is the sequence monotone? Why?

Solution. No, because it is neither increasing nor decreasing. ,

(c) Is the sequence bounded? Why?

Solution. Being bounded means it is both bounded above and below. So yes, clearly
there are bounds, i.e. −1 from below and 4, say. from above. ,

(d) The sequence is convergent. Find the limit.

Solution. Since it is convergent, the limit l exists; i.e. lim an = l exists. Hence,
n→∞

12 12
lim an+1 = lim ⇔ l= ⇔ l2 + 4l − 12 = 0 ⇔ l = −6, 2.
n→∞ n→∞ 4 + an 4+l
Since this is a sequence of positive numbers, l = 2. ,

[2+2+2+3=9 marks]

Useful Information

1. tan2 θ + 1 = sec2 θ
d
2. dx
[tan(x)] = sec2 x
d 1
3. dx
[tan−1 x] = 1+x2

(b − a)5 K4

(4)
4. |ES | ≤ , where K 4 = max f (x) : a ≤ x ≤ b
180n4
Z
R
p(x) dx 1
5. µ = e ,y= q(x)µ dx
µ

Set: 28 Jan 2019 Page 6 of 6 RSH20190130095600

Lautoka Campus

You might also like